Exercice 10-10
Soient \(f,g,h:\mathbb{R}\to\mathbb{R}\) telles que \[f(x)\leqslant g(x)\leqslant h(x)\qquad \forall x\in\mathbb{R}\,.\]
  1. Soit \(x_0\) un point tel que \(f(x_0)=h(x_0)\), et tel que \(f\) et \(h\) soient dérivables en \(x_0\), avec \[ f'(x_0)=h'(x_0)=m\,. \] Montrer que \(g\) est dérivable en \(x_0\), et que \(g'(x_0)=m\).
  2. Utiliser le point précédent pour montrer que la fonction \[ g(x)= \begin{cases} x^2\lfloor\frac{1}{x}\rfloor&\text{ si }x\neq 0\,,\\ 0&\text{ si }x=0 \end{cases} \] est dérivable en \(x_0=0\), et donner la valeur de \(g'(0)\).

Rappelons que \[ g'(x_0)=\lim_{x\to x_0}\frac{g(x)-g(x_0)}{x-x_0}\,. \]

trouver la valeur de \(g(x_0)\).

On utilisera alors une approche ''gendarmes'' pour étudier la limite du quotient \[ \frac{g(x)-g(x_0)}{x-x_0}\,. \]

Ce quotient pourra se majorer et se minorer différemment, à l'aide des quotients semblables pour \(f\) et \(h\), dépendant de si \(x\lt x_0\) ou \(x\gt x_0\).

Utiliser les majorations/minorations standards utilisées lorsqu'on étudie une valeur entière.

  1. Remarquons d'abord que \(f(x_0)=g(x_0)=h(x_0)\). Donc on peut écrire, pour tout \(x\), \[ f(x)-f(x_0)\leqslant g(x)-g(x_0)\leqslant h(x)-h(x_0)\,. \] On considère deux cas. D'une part, si \(x\gt x_0\), on peut diviser la double inégalité ci-dessus par \(x-x_0\gt 0\): \[ \frac{f(x)-f(x_0)}{x-x_0} \leqslant \frac{g(x)-g(x_0)}{x-x_0} \leqslant \frac{h(x)-h(x_0)}{x-x_0}\,, \] et donc, par le théorème des deux gendarmes (utilisé ici pour une limite latérale), \[ \lim_{x\to x_0^+} \frac{g(x)-g(x_0)}{x-x_0}=f'(x_0)=h'(x_0)=m\,. \] D'autre part, si \(x\lt x_0\), on peut diviser la double inégalité ci-dessus par \(x-x_0\lt 0\): \[ \frac{f(x)-f(x_0)}{x-x_0} \geqslant \frac{g(x)-g(x_0)}{x-x_0} \geqslant \frac{h(x)-h(x_0)}{x-x_0}\,, \] et donc \[ \lim_{x\to x_0^-} \frac{g(x)-g(x_0)}{x-x_0}=f'(x_0)=h'(x_0)=m\,. \] On a donc montré que \(g\) est dérivable en \(x_0\), et que \(g'(x_0)=m\).
  2. Puisque pour tout \(x\neq 0\), \[ \frac{1}{x}-1\leqslant \left\lfloor\frac{1}{x}\right\rfloor \leqslant \frac{1}{x}\,, \] on a \[ x-x^2\leqslant g(x) \leqslant x\qquad \forall x\in\mathbb{R}\,. \] Comme \(f(x)=x-x^2\) et \(h(x)=x\) sont toutes deux dérivables en zéro et leurs dérivées sont égales à \(1\), on conclut par la première partie de l'exercice que \(g\) est dérivable en \(0\) et \(g'(0)=1\).
    Remarquons que \(g\) est dérivable en zéro, mais qu'il existe des points arbitrairement proches de zéro où elle est discontinue.